Σελίδα 1 από 1

Ευκολότερη εκδοχή!

Δημοσιεύτηκε: Τετ Ιαν 08, 2020 11:08 pm
από matha
Με αφορμή αυτό το θέμα:

Να βρεθεί η μέγιστη τιμή του ακεραίου \displaystyle{m}, ώστε να ισχύει

\displaystyle{a^3+b^3+c^3-3abc\geq m(a-b)(b-c)(c-a)}

για κάθε \displaystyle{a,b,c\geq 0.}

Re: Ευκολότερη εκδοχή!

Δημοσιεύτηκε: Πέμ Ιαν 09, 2020 12:04 am
από Ορέστης Λιγνός
Η μέγιστη τιμή του m είναι 4.

Αν a=3,b=0,c=1, έχουμε 28 \geqslant 6m \Rightarrow m \leqslant 14/3, και αφού m φυσικός, είναι m \leqslant 4.

Μένει να δείξω ότι για m=4 η ανισότητα όντως ισχύει.

Θέλω δηλαδή να δείξω ότι a^3+b^3+c^3-3abc \geqslant 4(a-b)(b-c)(c-a).

Το αριστερό μέλος της παραπάνω είναι προφανώς \geqslant 0 από την ΑΜ-ΓΜ, οπότε αν (a-b)(b-c)(c-a) <0 δεν έχω κάτι να δείξω.

Έστω λοιπόν (a-b)(b-c)(c-a) \geqslant 0, και χωρίς βλάβη της γενικότητας, έστω a=\rm max\{a,b,c \}.

Τότε, a-b \geqslant 0 \geqslant c-a, οπότε b-c \leqslant 0 \Rightarrow a \geqslant c \geqslant b.

Έστω, a=b+k,c=b+\ell, με k,\ell \geqslant 0.

Τότε, θέλω να δείξω ότι (b+k)^3+b^3+(b+\ell)^3-3(b+k)(b+\ell)b \geqslant 4k\ell(k-\ell), που γράφεται (3k^2-3k\ell+3\ell^2)b+k^3+\ell^3 \geqslant 4k \ell(k-\ell).

Όμως, είναι προφανές ότι 3k^2-3k\ell+3\ell^2=3(k-\ell/2)^2+3\ell^2/4 \geqslant 0, άρα αρκεί να δείξω ότι k^3+\ell^3 \geqslant 4k \ell(k-\ell).

Θέτω k/\ell=m (αν \ell=0, έχω να δείξω ότι k^3 \geqslant 0, που είναι προφανής), και αρκεί να δείξω ότι m^3+1 \geqslant 4m(m-1) \Rightarrow m(m-2)^2+1 \geqslant 0, που ισχύει.

Άρα όντως η μέγιστη τιμή του m είναι 4.